Paradoja de la diferencia de potencial inducida

Entonces, estaba estudiando algunas notas de conferencias del programa de cursos abiertos del MIT, y me topé con este ejemploingrese la descripción de la imagen aquí

El ejemplo dice: El solenoide es tan largo que su campo magnético externo es insignificante. Su sección transversal es de 20 cm^2 de área, y el campo interior está hacia la derecha y aumenta a razón de 10^-2 T/s. Dos voltímetros idénticos se conectan como se muestra en los puntos A y C de la espira que encierra al solenoide y contiene las dos resistencias de 50 Ohm cada una. Esto nos da las lecturas de VM1=-10μV y VM2=10μV (los cálculos se mostrarán después). Ahora, esto es un poco raro, ya que los dos voltímetros nos dan diferentes medidas para el mismo punto, pero todo esto está incluido en el teoría de la inducción debido al cambio de flujo magnético (la integral de E.dS depende de la ruta real). Las cosas se vuelven más extrañas en esta situación: quiero cambiar las resistencias para estudiar qué cambia en las lecturas del voltímetro. Hago R1 = 40Ω y R2=60Ω.
Esto nos da: ingrese la descripción de la imagen aquí
Para VM1, consideramos el lazo que encierra a R1 y VM1 y nos da una lectura de IR1=0.2*40=8μV y el primer voltímetro muestra -8μV porque el voltaje en A es mayor que el de C (como que se muestra en la imagen) debido a la dirección del flujo de la corriente. Pero VM2 nos da IR2=-12μV (el signo negativo debido nuevamente al flujo).
Ahora, ¡esto no puedo entenderlo! Medimos los mismos puntos, ¡pero tenemos resultados diferentes! ¡Una cosa es obtener un signo opuesto, pero otra cosa es obtener dos números totalmente diferentes!
¿Alguien puede explicarme esto?

Hay una conferencia del profesor Lewin que está en línea, y antes hubo una pregunta al respecto. Veré si puedo encontrar el enlace; creo que esto es esencialmente un duplicado de ese.
Vea esta respuesta anterior y los enlaces en ella. Todavía no he encontrado el original que estoy buscando. Pero sí, es Lewin...
Veré la conferencia de Lewin primero. Podría ayudar.
Tiene que ver con el flujo encerrado en el bucle. Hice una pregunta sobre un tema similar anteriormente physics.stackexchange.com/questions/169076/… Mira mi comentario sobre la respuesta de Floris.

Respuestas (1)

La clave de esta pregunta es la posición del voltímetro y el flujo que contiene. Considere el siguiente diagrama:ingrese la descripción de la imagen aquí

Digamos que estamos encontrando el voltaje entre A y D usando el voltímetro METRO 1 . Podemos hacer esto usando uno de dos bucles:

  1. Uno que va directamente de A a D.ingrese la descripción de la imagen aquí
  2. El que va de A a B a C y luego a D.ingrese la descripción de la imagen aquí

El primero dará una respuesta de V 1 = I R 1 dado que no hay flujo que pase por el circuito que estamos considerando (que incluye M1 y R 1 ). Cuando lo hacemos alrededor del otro bucle, debemos incluir la fem inducida. Entonces, usando la ley de voltaje de Kirchoff alrededor de este bucle [ METRO 1 y R 2 ] (lo cual es válido si incluimos la fem inducida). Entonces obtenemos:

V 1 + I R 2 = ϵ
pero ϵ = I ( R 1 + R 2 ) entonces:
V 1 = I R 1
obtenemos la respuesta que obtuvimos antes. Se puede hacer un proceso análogo para el voltímetro en el segundo lado. Tal que:
V 2 = I R 2
[ V 1 denota la lectura en el voltímetro METRO 1 y V 2 eso en METRO 2 ] El punto es cuando tenemos campos no conservativos y al indicar la diferencia de voltaje entre dos puntos también debemos indicar dónde está posicionado el voltímetro, la respuesta depende de su ubicación, debido a la fem inducida. Tenga en cuenta que para cualquier ciclo que damos vueltas por una ubicación dada del voltímetro, todos están de acuerdo en la ubicación (sería muy extraño si obtuviéramos una contradicción aquí). Espero que esto responda tu pregunta.

[Ps Perdón por los diagramas pobres, el círculo en el medio se supone que es un solinodo.]

Su ejemplo trata sobre cómo medir la diferencia de voltaje de diferentes maneras. Pero siempre da el mismo ejemplo. Mi problema es cuando dos voltímetros miden una diferencia de voltaje diferente para los mismos dos puntos. No sé si no entiendo algo que usted están tratando de decir. Ahora estoy viendo el video que me dijiste en los comentarios.
El punto es para un campo conservador, la diferencia de voltaje está mal definida. El circuito con el que está midiendo el voltaje (es decir, el que contiene el voltímetro) también se ve afectado por la fem inducida en el circuito. Mi respuesta muestra por qué obtienes los valores que obtienes. La razón por la que con diferentes voltímetros obtienes diferentes respuestas es porque cada uno se ve afectado de manera diferente por el flujo que pasa a través del bucle.
Entiendo lo que dice, pero no puedo ver en su ejemplo dónde los dos voltímetros miden la diferencia de potencial entre los mismos dos puntos pero tienen lecturas diferentes
Tienen diferentes lecturas ya que para METRO 2 el bucle pasando R 2 no contiene flujo mientras que a través de R 1 hace. El flujo a través del circuito cambia el voltaje medido a través de las dos terminales. Es al revés para METRO 1 el bucle a través R 1 no tiene flujo a través de él, mientras que a través de R 2 hace.
Entonces, para el punto A y D, ¿M1 mide V=-IR1 mientras que M2 mide V=IR2?
Sí [los puntos A y B son equivalentes, podrían haber sido dibujados en el mismo lugar. lamiendo con D y C ]
Ok, ahora entiendo. Pero, ¿cuál es la intuición detrás de esto? Los voltímetros nos dan diferentes potenciales, entonces, ¿cuál es el verdadero? Quiero decir, ¡estamos hablando de energía por carga! ¿Cómo podemos obtener dos resultados diferentes? Las matemáticas lo demuestran , pero ¿cómo podemos pensar en ello?
Para los bucles más grandes (el que contiene el flujo), el electrón recibe un 'cierre' resultante adicional, que aumenta o disminuye su energía según la dirección en la que se encuentre. Esto no ocurre en el camino que no contiene flujo. Por lo tanto, debemos incluir esta energía adicional en el camino que contiene flujo, pero no en el que no contiene flujo.
Como lo entendí de la conferencia de Lewin, es como Kirchoff, pero ahora si integras alrededor del circuito (bucle cerrado) no obtienes cero (la energía no se conserva) pero obtienes la fem inducida